Tải bản đầy đủ (.pdf) (82 trang)

Một số dạng bất đẳng thức phân thức có ràng buộc bởi đa thức Viète

Bạn đang xem bản rút gọn của tài liệu. Xem và tải ngay bản đầy đủ của tài liệu tại đây (496.7 KB, 82 trang )

ĐẠI HỌC THÁI NGUYÊN
TRƯỜNG ĐẠI HỌC KHOA HỌC
LÊ THỊ MAI
MỘT SỐ DẠNG BẤT ĐẲNG THỨC PHÂN THỨC
CÓ RÀNG BUỘC BỞI ĐA THỨC VIÈTE
LUẬN VĂN THẠC SĨ TOÁN HỌC
Thái Nguyên - 2015
ĐẠI HỌC THÁI NGUYÊN
TRƯỜNG ĐẠI HỌC KHOA HỌC
LÊ THỊ MAI
MỘT SỐ DẠNG BẤT ĐẲNG THỨC PHÂN THỨC
CÓ RÀNG BUỘC BỞI ĐA THỨC VIÈTE
Chuyên ngành: Phương pháp Toán sơ cấp
Mã số: 60 46 01 13
LUẬN VĂN THẠC SĨ TOÁN HỌC
Người hướng dẫn khoa học:
GS.TSKH. NGUYỄN VĂN MẬU
Thái Nguyên - 2015
i
Mục lục
Lời cam đoan iii
Mở đầu 1
1 Một số kiến thức bổ trợ 3
1.1 Đa thức đối xứng ba biến . . . . . . . . . . . . . . . . . . . . . . . 3
1.2 Một số dạng bất đẳng thức cổ điển . . . . . . . . . . . . . . . . . . 4
1.2.1 Bất đẳng thức AM-GM . . . . . . . . . . . . . . . . . . . . 4
1.2.2 Bất đẳng thức Cauchy - Schwarz . . . . . . . . . . . . . . . 5
1.2.3 Bất đẳng thức Karamata . . . . . . . . . . . . . . . . . . . 6
2 Một số dạng bất đẳng thức phân thức có ràng buộc bởi đa thức Viète 7
2.1 Bất đẳng thức có tổng không đổi với hàm phân thức hữu tỉ . . . . . 7
2.1.1 Sử dụng bất đẳng thức AM-GM . . . . . . . . . . . . . . . 7


2.1.2 Sử dụng bất đẳng thức Cauchy-Schwarz . . . . . . . . . . . 13
2.2 Bất đẳng thức có tích không đổi với hàm phân thức . . . . . . . . . 22
2.2.1 Sử dụng bất đẳng thức AM-GM . . . . . . . . . . . . . . . 22
2.2.2 Sử dụng bất đẳng thức Cauchy-Schwarz . . . . . . . . . . . 28
2.3 Một số bài toán liên quan . . . . . . . . . . . . . . . . . . . . . . . 32
3 Một số phương pháp khảo sát bất đẳng thức dạng phân thức 39
3.1 Bất đẳng thức phân thức sinh bởi tam thức bậc hai trên một khoảng . 39
3.2 Bất đẳng thức sinh bởi hàm phân tuyến tính trên một khoảng . . . . 43
3.3 Phương pháp nội suy bất đẳng thức . . . . . . . . . . . . . . . . . . 46
ii
3.4 Phương pháp tiếp tuyến . . . . . . . . . . . . . . . . . . . . . . . . 62
3.5 Phương pháp khảo sát hàm số . . . . . . . . . . . . . . . . . . . . . 67
Kết luận và Đề nghị 76
Tài liệu tham khảo 77
Danh mục các công trình khoa học liên quan đến luận văn 77
iii
Lời cam đoan
Tôi xin cam đoan các số liệu và kết quả nghiên cứu trong luận văn này là không
trùng lặp với các đề tài khác và được hoàn thành dưới sự hướng dẫn của GS. TSKH.
Nguyễn Văn Mậu. Một số kết quả trong luận văn là mới và chưa từng được ai công
bố trong bất cứ một công trình nào khác mà tôi biết. Tôi cũng xin cam đoan mọi
thông tin tr ích dẫn trong luận văn đã được chỉ rõ nguồn gốc.
Thái Nguyên, ngày 10 tháng 4 năm 2015
Học viên
Lê Thị Mai
1
Mở đầu
Bất đẳng thức là một nội dung chuyên đề quan trọng của Toán học. Ngay từ khi
ra đời, bất đẳng thức đã có sức hút mạnh mẽ đối với những người yêu toán, không
chỉ ở vẻ đẹp hình thức mà cả những bí ẩn nó mang đến, luôn thôi thúc người ta quan

tâm tìm tòi, sáng tạo. Đặc biệt, bất đẳng thức còn có nhiều ứng dụng trong các môn
khoa học khác và trong ứng dụng thực tế. Ngày nay, bất đẳng thức vẫn luôn chiếm
một vị trí quan trọng và vẫn thường xuất hiện trong các kì thi Olympic quốc gia, khu
vực và quốc tế.
Phân thức hữu tỷ là một trong những khái niệm cơ bản của chương trình Toán
ở bậc học phổ thông. Đặc biệt, ở các trường THPT chuyên và các lớp chuyên toán
có rất nhiều dạng toán liên quan đến hàm phân thức. Trong các kỳ thi học sinh giỏi
Toán trong nước và các kỳ thi Olympic Toán của các nước trên thế giới, có nhiều bài
toán về dãy số, bất đẳng thức, phương trình, bất phương trình và hệ bất phương trình
sinh bởi các hàm số dạng phân thức và vì thế cần biết cách giải vận dụng tính đặc
thù của biểu thức phân thức đã cho. Hiện nay các tài liệu có tính hệ thống về vấn đề
này còn chưa được đề cập nhiều.
Là một giáo viên THPT, tôi muốn nghiên cứu sâu hơn về bất đẳng thức nhằm
nâng cao chuyên môn phục vụ cho quá trình giảng dạy và bồi dưỡng học sinh giỏi,
vậy nên tôi đã chọn đề tài "Một số dạng bất đẳng thức phân thức có ràng buộc bởi đa
thức Viète” làm luận văn thạc sĩ của mình.
Bất đẳng thức vô cùng rộng lớn, trong thời gian ngắn, tôi chỉ có thể khảo sát một
số chuyên đề nhỏ trong đó. Dưới sự hướng dẫn của GS. TSKH Nguyễn Văn Mậu, tác
giả đã hoàn thành luận văn với để tài
2
Một số dạng bất đẳng thức phân thức có ràng buộc bởi đa thức Viète
Luận văn được chia làm ba chương:
• Chương 1. Một số kiến thức bổ trợ.
• Chương 2. Một số dạng bất đẳng thức phân thức có ràng buộc bởi đa thức
Viète.
• Chương 3. Một số phương pháp khảo sát bất đẳng thức dạng phân thức.
Mặc dù có nhiều cố gắng, song do thời gian và trình độ bản thân còn hạn chế nên
luận văn khó tránh khỏi những thiếu sót. Vì vậy, tác giả rất mong nhận được sự góp
ý của các thầy cô, các anh chị đồng nghiệp và các bạn để luận văn được hoàn thiện
hơn.

Qua luận văn này, tác giả xin bày tỏ lòng biết ơn sâu sắc đến GS.TSKH Nguyễn
Văn Mậu, người Thầy đã truyền cho tác giả có niềm say mê nghiên cứu toán học.
Thầy đã tận tình hướng dẫn, giúp đỡ tác giả trong suốt quá trình học tập và hoàn
thiện luận văn này.
Tác giả xin chân thành cảm ơn Ban giám hiệu, Phòng Đào tạo, Khoa Toán và các
thầy cô đã tạo điều kiện thuận lợi cho em hoàn thành bản luận văn này.
Em xin chân thành cảm ơn!
Thái Nguyên, ngày 26 tháng 03 năm 2015
Lê Thị Mai
Học viên Cao học Toán Lớp B, khóa 06/2013-06/2015
Chuyên ngành Phương pháp Toán sơ cấp
Trường Đại học Khoa học - Đại học Thái Nguyên
Email:
3
Chương 1
Một số kiến thức bổ trợ
Chương này trình bày một số tính chất của đa thức cần thiết để sử dụng trong các
chương sau, dựa theo các tài liệu [1]-[5] và trình bày một số dạng bất đẳng thức cổ
điển được sử dụng nhiều trong các chương sau như Bất đẳng thức AM - GM, Bất
đẳng thức Cauchy - schwarz, bất đẳng thức Karamata, . . .
1.1 Đa thức đối xứng ba biến
Định nghĩa 1.1. Một đơn thức ϕ(x, y, z) của các biến x, y, z được hiểu là hàm số có
dạng
ϕ(x, y, z) = a
klm
x
k
y
l
z

m
,
trong đó k, l, m ∈ N được gọi là bậc của biến x, y, z, số a
klm
∈ R

= R\{0} được
gọi là hệ số của đơn thức, còn số k + l + m được gọi là bậc của đơn thức ϕ(x, y, z).
Định nghĩa 1.2. Một hàm số P (x, y, z) của các biến x, y, z được gọi là một đa thức
nếu nó có thể được biểu diễn ở dạng tổng hữu hạn các đơn thức
P (x, y, z) =

k,l,m∈N
k+l+m=n
a
klm
x
k
y
l
z
m
, n ∈ N.
Bậc lớn nhất của các đơn thức trong đa thức được gọi là bậc của đa thức.
Định nghĩa 1.3. Đa thức P (x, y, z) được gọi là đối xứng, nếu nó không thay đổi với
mọi hoán vị của x, y, z, nghĩa là
P (x, y, z) = P(y, x, z) = P (z, y, x) = P (x, z, y).
4
Định nghĩa 1.4. Đa thức f (x, y, z) được gọi là thuần nhất bậc m, nếu
f(tx, ty, tz) = t

m
f(x, y, z), t = 0
Định nghĩa 1.5. Các đa thức
σ
1
= x + y + z, σ
2
= xy + yz + zx, σ
3
= xyz,
được gọi là đa thức đối xứng cơ sở của các biến x, y, z.
Định nghĩa 1.6 (Tổng lũy thừa). Các đa thức s
k
= x
k
+ y
k
+ z
k
, (k = 0, 1, ), được
gọi là tổng lũy thừa bậc k của các biến x, y, z.
Tính chất 1.1 (Công thức Newton). Với mọi k ∈ Z, ta có hệ thức
s
k
= σ
1
s
k−1
− σ
2

s
k−2
+ σ
3
s
k−3
.
Tính chất 1.2. Một tổng lũy thừa s
k
= x
k
+ y
k
+ z
k
đều có thể biểu diễn được dưới
dạng một đa thức theo các biến σ
1
, σ
2
, σ
3
.
Định lí 1.1 (Công thức Waring). Tổng lũy thừa s
k
được biểu diễn qua các đa thức
đối xứng cở sở theo công thức
s
k
k

=

0≤l,m,n
l+2m+3n=k
(−1)
k−l−m−n
(l + m + n − 1)!
l!m!n!
σ
l
1
σ
m
2
σ
n
3
.
1.2 Một số dạng bất đẳng thức cổ điển
1.2.1 Bất đẳng thức AM-GM
Định lí 1.2 (Xem [3]-[4]). Giả sử a
1
, a
2
, . . . , a
n
là các số thực không âm, khi đó ta
luôn có
a
1

+ a
2
+ ··· + a
n
n

n

a
1
a
2
. . . a
n
.
Đẳng thức xảy ra khi và chỉ khi a
1
= a
2
= ··· = a
n
.
5
Hệ quả 1.1. Với mọi số thực dương a
1
, a
2
, . . . , a
n
, ta có


1
a
1
+
1
a
2
+ ··· +
1
a
n

(a
1
+ a
2
+ ··· + a
n
) ≥ n
2
.
Đẳng thức xảy ra khi và chỉ khi a
1
= a
2
= ··· = a
n
.
Hệ quả 1.2. Với mọi số thực a, b, c, ta luôn có

1. a
2
+ b
2
+ c
2
≥ ab + bc + ca
2. a
2
+ b
2
+ c
2

(a + b + c)
2
3
3. (a + b + c)
2
≥ 3(ab + bc + ca)
4. a
2
b
2
+ b
2
c
2
+ c
2

a
2
≥ abc(a + b + c)
5. (ab + bc + ca)
2
≥ 3abc(a + b + c).
1.2.2 Bất đẳng thức Cauchy - Schwarz
Định lí 1.3 (Xem [3]-[4]). Nếu a
1
, a
2
, . . . , a
n
, b
1
, b
2
, . . . , b
n
là các số thực tùy ý thì
(a
1
b
1
+ a
2
b
2
+ ··· + a
n

b
n
)
2


a
2
1
+ a
2
2
+ ··· + a
2
n

b
2
1
+ b
2
2
+ ··· + b
2
n

. (1.1)
Đẳng thức xảy ra khi và chỉ khi
a
1

b
1
=
a
2
b
2
= ··· =
a
n
b
n
(ở đây ta sử dụng quy ước nếu
mẫu bằng 0 thì tử cũng bằng 0).
Nhận xét 1.1. Theo bất đẳng thức (1.1) chọn a
i
=
x
i

y
i
và b
i
=

y
i
với x
i

, y
i

R, y
i
> 0. Ta thu được bất đẳng thức Cauchy-Schwarz dạng phân thức (hay còn gọi
là bất đẳng thức Cauchy-Schwarz dạng Engel).
Hệ quả 1.3. Nếu x
1
, x
2
, . . . , x
n
là các số thực và y
1
, y
2
, . . . , y
n
là các số thực dương
thì
x
2
1
y
1
+
x
2
2

y
2
+ ··· +
x
2
n
y
n

(x
1
+ x
2
+ . . . x
n
)
2
y
1
+ y
2
+ ··· + y
n
.
Đẳng thức xảy ra khi và chỉ khi
x
1
y
1
=

x
2
y
2
= ··· =
x
n
y
n
.
6
1.2.3 Bất đẳng thức Karamata
Định lí 1.4 (Bất đẳng thức Karamata). Cho hai dãy số {x
k
, y
k
∈ (a, b), k = 1, 2, . . . , n},
thỏa mãn điều kiện
x
1
≥ x
2
≥ ··· ≥ x
n
, y
1
≥ y
2
≥ ··· ≥ y
n





























x
1

≥ y
1
x
1
+ x
2
≥ y
1
+ y
2

x
1
+ x
2
+ ··· + x
n−1
≥ y
1
+ y
2
+ ··· + y
n−1
x
1
+ x
2
+ ··· + x
n
= y

1
+ y
2
+ ··· + y
n
Khi đó, ứng với mọi hàm số lồi khả vi f(x)(f

(x) ≥ 0) trên (a, b), ta đều có
f(x
1
) + f(x
2
) + ··· + f(x
n
) ≥ f(y
1
) + f(y
2
) + ··· + f(y
n
).
Đẳng thức xảy ra khi x
i
= y
i
, i = 1, 2, . . . n.
Ta cũng phát biểu tương tự đối với hàm số lõm bằng cách đổi chiều dấu bất đẳng
thức.
Định lí 1.5. Cho hàm số y = f(x) liên tục và có đạo hàm cấp 2 trên (a; b).
a. Nếu f


(x) ≥ 0, ∀x ∈ (a; b) thì f(x) ≥ f

(x
0
)(x−x
0
)+f (x
0
), ∀x, x
0
∈ (a; b).
b. Nếu f

(x) ≤ 0, ∀x ∈ (a; b) thì f(x) ≤ f

(x
0
)(x−x
0
)+f(x
0
), ∀x, x
0
∈ (a; b).
Đẳng thức trong hai bất đẳng thức trên xảy ra khi x = x
0
.
7
Chương 2

Một số dạng bất đẳng thức phân thức có ràng
buộc bởi đa thức Viète
Trong mục này chúng ta thường sử dụng bất đẳng thức trung gian sau đây.
Bài toán 2.1. Với a, b, c là những số thức dương. Chứng minh rằng
a
m+n
+ b
m+n
+ c
m+n
≥ a
m
b
n
+ b
m
c
n
+ c
m
a
n
.
Lời giải. Ta có
ma
m+n
+ nb
m+n
m + n
≥ a

m
b
n
(bất đẳng thức AM-GM với m + n số),
mb
m+n
+ nc
m+n
m + n
 b
m
c
n
,
mc
m+n
+ na
m+n
m + n
 c
m
a
n
.
Cộng các vế tương ứng của ba bất đẳng thức trên, ta thu được bất đẳng thức cần
chứng minh.
2.1 Bất đẳng thức có tổng không đổi với hàm phân thức
hữu tỉ
2.1.1 Sử dụng bất đẳng thức AM-GM
Đối với bất đẳng thức P (x, y, z) ≥ 0 (≤ 0), trong đó P (x, y, z) là phân thức hữu

tỉ và có tổng x + y + z không đổi, thì khi đó sử dụng các kĩ thuật của bất đẳng thức
AM −GM như dự đoán dấu bằng xảy ra, kĩ thuật Cauchy ngược dấu, đặt ẩn phụ,
tỏ ra rất hiệu quả.
8
Bài toán 2.2. Với a, b, c là các số thực dương thỏa mãn a + b + c = 3. Chứng minh
rằng
a
2
b + 2
+
b
2
c + 2
+
c
2
a + 2
≥ 1.
Lời giải. Áp dụng bất đẳng thức AM-GM, ta có
a
2
b + 2
+
b + 2
9

2a
3
b
2

c + 2
+
c + 2
9

2b
3
c
2
a + 2
+
a + 2
9

2c
3
.
Cộng các bất đẳng thức cùng chiều ta được
a
2
b + 2
+
b
2
c + 2
+
c
2
a + 2


5
9
(a + b + c) −
2
3
= 1.
Đẳng thức xảy ra khi và chỉ khi a = b = c = 1.
Nhận xét 2.1. Khi sử dụng bất đẳng thức AM - GM, ta cần chú ý
1. Lựa chọn thừa số để đảm bảo dấu đẳng thức của bất đẳng thức xảy ra,
2. Bổ sung thêm một số số hạng để sau khi sử dụng bất đẳng thức AM - GM ta
khử được mẫu số của biểu thức phân số.
Một số ví dụ cùng dạng
Bài toán 2.3. Với a, b, c là các số thực dương thỏa mãn a + b + c = 3. Chứng minh
rằng
1
a(a + b)
+
1
b(b + c)
+
1
c(c + a)

3
2
.
Bài toán 2.4. Với a, b, c là các số thực dương thỏa mãn a + b + c = 3. Chứng minh
rằng
a
3

b(b + c)
+
b
3
c(c + a)
+
c
3
a(a + b)

3
4
.
Bài toán 2.5. Với a, b, c là các số thực dương thỏa mãn a + b + c = 3. Chứng minh
rằng
a
3
(a + b)(b + c)
+
b
3
(b + c)(c + a)
+
c
3
(c + a)(a + b)

3
4
.

9
Nhận xét 2.2. Một trong những kĩ thuật hay, khéo léo, mới mẻ và ấn tượng nhất của
bất đẳng thức AM - GM là kĩ thuật Cauchy ngược dấu.
Bài toán 2.6 (Bulgaria TST 2003). Với ba số thực dương a, b, c thỏa mãn điều kiện
a + b + c = 3. Chứng minh rằng
a
1 + b
2
+
b
1 + c
2
+
c
1 + a
2

3
2
.
Lời giải. Áp dụng bất đẳng thức AM-GM, ta có
a
1 + b
2
= a −
ab
2
1 + b
2
≥ a −

ab
2
2b
= a −
ab
2
.
Tương tự, ta có
b
1 + c
2
≥ b −
bc
2
c
a
2
+ 1
≥ c −
ac
2
.
Cộng vế với vế, ta có
a
1 + b
2
+
b
1 + c
2

+
c
1 + a
2
≥ a + b + c −
1
2
(ab + bc + ca) = 3 −
ab + bc + ca
2
.
Mặt khác, ta có
ab + bc + ca ≤
(a + b + c)
2
3
= 3,
do đó
a
1 + b
2
+
b
1 + c
2
+
c
1 + a
2
≥ 3 −

ab + bc + ca
2

3
2
.
Đẳng thức xảy ra khi và chỉ khi a = b = c = 1.
Tiếp theo, ta có một số bài toán cùng dạng.
Bài toán 2.7. Với a, b, c là các số thực dương thỏa mãn a + b + c = 5. Chứng minh
rằng
a
3
a
2
+ b
2
+
b
3
b
2
+ c
2
+
c
3
c
2
+ a
2


5
2
.
Bài toán 2.8. Với a, b, c là các số thực dương thỏa mãn a + b + c = 3. Chứng minh
rằng
a + 1
1 + b
2
+
b + 1
1 + c
2
+
c + 1
1 + a
2
≥ 3.
10
Bài toán 2.9. Với a, b, c là các số thực dương thỏa mãn a + b + c = 3. Chứng minh
rằng
a
2
a + 2b
2
+
b
2
b + 2c
2

+
c
2
c + 2a
2
≥ 1.
Bài toán 2.10. Với a, b, c là các số thực dương thỏa mãn a + b + c = 3. Chứng minh
rằng
a
2
a + 2b
3
+
b
2
b + 2c
3
+
c
2
c + 2a
3
≥ 1.
Bài toán 2.11. Với a, b, c là các số thực dương thỏa mãn a + b + c = 3. Chứng minh
rằng
2(ab + bc + ca) +
1
ab
+
1

bc
+
1
ca
≥ 9.
Lời giải. Cộng cả hai vế của bất đẳng thức cần chứng minh với a
2
+ b
2
+ c
2
, ta được
a
2
+ b
2
+ c
2
+ 2(ab + bc + ca) +
1
ab
+
1
bc
+
1
ca
≥ 9 + a
2
+ b

2
+ c
2
⇔(a + b + c)
2
+
1
ab
+
1
bc
+
1
ca
≥ 9 + a
2
+ b
2
+ c
2

1
ab
+
1
bc
+
1
ca
≥ a

2
+ b
2
+ c
2

a + b + c
abc
≥ a
2
+ b
2
+ c
2

3
abc
≥ a
2
+ b
2
+ c
2
⇔abc(a
2
+ b
2
+ c
2
) ≤ 3

⇔abc(a + b + c)(a
2
+ b
2
+ c
2
) ≤ 9.
Ta có
abc(a + b + c)(a
2
+ b
2
+ c
2
) ≤
(ab + bc + ca)
2
3
(a
2
+ b
2
+ c
2
).
Do đó ta chứng minh
(ab + bc + ca)
2
3
(a

2
+ b
2
+ c
2
) ≤ 9 ⇔ (ab + bc + ca)
2
(a
2
+ b
2
+ c
2
) ≤ 27.
Áp dụng bất đẳng thức AM-GM, ta có
(ab + bc + ca)
2
(a
2
+ b
2
+ c
2
) ≤

(ab + bc + ca) + (ab + bc + ca) + a
2
+ b
2
+ c

2
3

3
=

(a + b + c)
2
3

3
= 27.
11
Vậy bất đẳng thức được chứng minh.
Đẳng thức xảy ra khi và chỉ khi a = b = c = 1.
Bài toán 2.12 (Turkey TST 2007). Với a, b, c là các số thực dương thỏa mãn a + b +
c = 1. Chứng minh rằng
1
ab + 2c
2
+ 2c
+
1
bc + 2a
2
+ 2a
+
1
ca + 2b
2

+ 2b

1
ab + bc + ca
.
Lời giải. Áp dụng bất đẳng thức AM-GM, ta có
ab + 2c
2
+ 2c = ab + 2c
2
+ 2c(a + b + c)
= ab + 4c
2
+ 2ac + 2bc
= (b + 2c)(a + 2c)
=
a(b + 2c)b(a + 2c)
ab
=
(ab + 2ac)(ab + 2bc)
ab

[ab + 2ac + ab + 2bc]
2
4ab
=
(ab + bc + ca)
2
ab
từ đó suy ra

1
ab + 2c
2
+ 2c

ab
(ab + bc + ca)
2
.
Tương tự, ta có
1
bc + 2a
2
+ 2a

bc
(ab + bc + ca)
2
1
ca + 2b
2
+ 2b

ca
(ab + bc + ca)
2
.
Cộng các bất đẳng thức cùng chiều, ta có
1
ab + 2c

2
+ 2c
+
1
bc + 2a
2
+ 2a
+
1
ca + 2b
2
+ 2b

1
ab + bc + ca
.
Đẳng thức xảy ra khi và chỉ khi a = b = c =
1
3
.
12
Bài toán 2.13. Với các số thực dương a, b, c thỏa mãn a + b + c = 3. Chứng minh
rằng
a
2
+ bc
b + ca
+
b
2

+ ca
c + ab
+
c
2
+ ab
a + bc
≥ 3.
Lời giải. Bất đẳng thức cần chứng minh tương đương với
a
2
+ bc
3b + 3ca
+
b
2
+ ca
3c + 3ab
+
c
2
+ ab
3a + 3bc
≥ 1

a
2
+ bc
b(a + b + c) + 3ca
+

b
2
+ ca
c(a + b + c) + 3ab
+
c
2
+ ab
a(a + b + c) + 3bc
≥ 1.
Mặt khác
3b+3ac = b(a+b+c)+3ac ≤ b(a+b+c)+ac+a
2
+c
2
= a
2
+b
2
+c
2
+ab+bc+ca
do đó
a
2
+ bc
3b + 3ac

a
2

+ bc
a
2
+ b
2
+ c
2
+ ab + bc + ca
.
Tương tự, ta có
b
2
+ ca
3c + 3ab

b
2
+ ca
a
2
+ b
2
+ c
2
+ ab + bc + ca
c
2
+ ab
3a + 3bc


c
2
+ ab
a
2
+ b
2
+ c
2
+ ab + bc + ca
.
Cộng các bất đẳng thức cùng chiều, ta có
a
2
+ bc
3b + 3ca
+
b
2
+ ca
3c + 3ab
+
c
2
+ ab
3a + 3bc
≥ 1.
Đẳng thức xảy ra khi và chỉ khi a = b = c = 1.
Bài toán 2.14. Với các số thực dương thỏa mãn a + b + c = 3. Chứng minh rằng
ab


c
2
+ 3
+
bc

a
2
+ 3
+
ca

b
2
+ 3

3
2
.
Lời giải. Ta luôn có
(a + b + c)
2
3
≥ ab + bc + ca
⇔ 3 ≥ ab + bc + ca.
13
Suy ra
ab


c
2
+ 3

ab

c
2
+ ab + bc + ca
=
ab

(a + b) (a + c)
.
Từ đó áp dụng bất đẳng thức AM-GM, ta có
ab

(b + c) (c + a)

1
2

ab
b + c
+
ab
c + a

.
Chứng minh tương tự

bc

a
2
+ 3

bc

(a + c) (a + b)

1
2

bc
c + a
+
bc
a + b

ca

b
2
+ 3

ca

(b + a) (b + c)

1

2

ca
b + a
+
ca
b + c

.
Cộng các bất đẳng thức cùng chiều, ta có
ab

c
2
+ 3
+
bc

a
2
+ 3
+
ca

b
2
+ 3

1
2


ab
b + c
+
ca
b + c

+

ab
c + a
+
bc
c + a

+

bc
a + b
+
ca
a + b

=
a + b + c
3
=
3
2
.

Đẳng thức xảy ra khi và chỉ khi a = b = c = 1.
2.1.2 Sử dụng bất đẳng thức Cauchy-Schwarz
Nếu yêu cầu của bất đẳng thức là chứng minh P (x, y, z) ≥ 0(≤ 0) với P(x, y, z)
có dạng tổng các bình phương hoặc phân thức với tử số của mỗi phân thức có dạng
bình phương như vậy ta có thể nghĩ đến việc áp dụng bất đẳng thức Cauchy- Schwarz.
Hoặc sử dụng giả thiết x + y + z không đổi ta có thể biến đổi để đưa bất đẳng thức
về dạng trên để sử dụng Cauchy-Schwarz.
Bài toán 2.15. Với các số thực dương a, b, c thỏa mãn điều kiện a + b + c = 1.
Chứng minh rằng
a
1 + b − a
+
b
1 + c − b
+
c
1 + a − c
≥ 1.
14
Lời giải. Ta luôn có
a
1 + b − a
+
b
1 + c − b
+
c
1 + a − c
=
a

2b + c
+
b
2c + a
+
c
2a + b
=
a
2
2ab + ac
+
b
2
2bc + ab
+
c
2
2ac + bc
.
Áp dụng bất đẳng thức Cauchy-Schwarz, ta có
a
2
2ab + ac
+
b
2
2bc + ab
+
c

2
2ac + bc

(a + b + c)
2
3 (ab + bc + ca)
.
Theo hệ quả của bất đẳng thức AM-GM, ta có
(a + b + c)
2
≥ 3(ab + bc + ca),
do đó
a
2
2ab + ac
+
b
2
2bc + ab
+
c
2
2ac + bc

(a + b + c)
2
3 (ab + bc + ca)
≥ 1.
Đẳng thức xảy ra khi và chỉ khi a = b = c = 1.
Bài toán 2.16. Với các số thực dương a, b, c sao cho a + b + c = 1. Chứng minh

1
a
2
+ b
2
+ c
2
+
1
ab
+
1
bc
+
1
ca
≥ 30.
Lời giải. Áp dụng bất đẳng thức Cauchy-Schwarz cho hai bộ số

1

a
2
+ b
2
+ c
2
,
1


ab
,
1

bc
,
1

cd




a
2
+ b
2
+ c
2
, 3

ab, 3

bc, 3

ca

.
Ta có


1
a
2
+ b
2
+ c
2
+
1
ab
+
1
bc
+
1
ca


a
2
+ b
2
+ c
2
+ 9ab + 9bc + 9ca

≥ (1 + 3 + 3 + 3)
2



1
a
2
+ b
2
+ c
2
+
1
ab
+
1
bc
+
1
ca


(a + b + c)
2
+ 7(ab + bc + ca)

≥ 100

1
a
2
+ b
2
+ c

2
+
1
ab
+
1
bc
+
1
ca

100
(a + b + c)
2
+ 7(ab + bc + ca)
.
15
Theo hệ quả của bất đẳng thức AM −GM, ta có
(a + b + c)
2
3
≥ ab + bc + ca
nên
1
a
2
+ b
2
+ c
2

+
1
ab
+
1
bc
+
1
ca

100
1 + 7
(a + b + c)
2
3
= 30.
Đẳng thức xảy ra khi và chỉ khi a = b = c =
1
3
.
Bài toán 2.17 (Iran MO TST 2009). Với các số thực dương a, b, c thỏa mãn điều
kiện a + b + c = 3. Chứng minh rằng
1
a
2
+ b
2
+ 2
+
1

b
2
+ c
2
+ 2
+
1
c
2
+ a
2
+ 2

3
4
.
Lời giải. Bất đẳng thức cần chứng minh


1
2

1
a
2
+ b
2
+ 2

+


1
2

1
b
2
+ c
2
+ 2

+

1
2

1
c
2
+ a
2
+ 2


3
2

3
4


a
2
+ b
2
a
2
+ b
2
+ 2
+
b
2
+ c
2
b
2
+ c
2
+ 2
+
c
2
+ a
2
c
2
+ a
2
+ 2


3
2

(a + b)
2
+ (a − b)
2
a
2
+ b
2
+ 2
+
(b + c)
2
+ (b − c)
2
b
2
+ c
2
+ 2
+
(c + a)
2
+ (c − a)
2
c
2
+ a

2
+ 2
≥ 3


(a + b)
2
a
2
+ b
2
+ 2
+
(b + c)
2
b
2
+ c
2
+ 2
+
(c + a)
2
c
2
+ a
2
+ 2

+


(a − b)
2
a
2
+ b
2
+ 2
+
(b − c)
2
b
2
+ c
2
+ 2
+
(c − a)
2
c
2
+ a
2
+ 2

≥ 3.
Theo bất đẳng thức Cauchy-Schwarz, ta có
(a + b)
2
a

2
+ b
2
+ 2
+
(b + c)
2
b
2
+ c
2
+ 2
+
(c + a)
2
c
2
+ a
2
+ 2

[2(a + b + c)]
2
2a
2
+ 2b
2
+ 2c
2
+ 6

(2.1)

(b − a)
2
a
2
+ b
2
+ 2
+
(b − c)
2
b
2
+ c
2
+ 2
+
(c − a)
2
c
2
+ a
2
+ 2

(2b − 2a)
2
2a
2

+ 2b
2
+ 2c
2
+ 6
. (2.2)
16
Bất đẳng thức được chứng minh nếu
4 (a + b + c)
2
+ (2b − 2a)
2
2a
2
+ 2b
2
+ 2c
2
+ 6
≥ 3
⇔ 2(a + b + c)
2
+ 2(b − a)
2
≥ 3(a
2
+ b
2
+ c
2

+ 3)
⇔ (a + b + c)
2
+ 2 (b − a)
2
≥ 3

a
2
+ b
2
+ c
2

⇔ bc − ab + ca − c
2
≥ 0
⇔ (c − a)(b − c) ≥ 0. (2.3)
Có thể nhận thấy bất đẳng thức (2.3) không phải luôn đúng nhưng ta có thể ép
nó đúng. Thật vậy, sử dụng các đánh giá tương tự (2.3), ta có
(a − b)
2
a
2
+ b
2
+ 2
+
(c − b)
2

b
2
+ c
2
+ 2
+
(c − a)
2
c
2
+ a
2
+ 2

(2c − 2b)
2
2a
2
+ 2b
2
+ 2c
2
+ 6
(a − b)
2
a
2
+ b
2
+ 2

+
(b − c)
2
b
2
+ c
2
+ 2
+
(a − c)
2
c
2
+ a
2
+ 2

(2a − 2c)
2
2a
2
+ 2b
2
+ 2c
2
+ 6
và cùng lần lượt đưa bài toán về chứng minh
(a − b)(c − a) ≥ 0 (2.4)
(b − c)(a − b) ≥ 0. (2.5)
Như vậy nếu trong các bất đẳng thức (2.3),(2.4) và (2.5) có một bất đẳng thức đúng

thì bài toán sẽ được chứng minh song. Ta thấy rằng
[(c − a)(b − c)] [(a − b)(c − a)] [(b −c)(a −b)] = (a −b)
2
(b − c)
2
(c − a)
2
≥ 0
nên ít nhất một trong ba số (c − a)(b − c), (a − b)(c −a), (b − c)(a −b) sẽ có một
số không âm. Tức là phải có ít nhất một bất đẳng thức đúng hay bất đẳng thức được
chứng minh.
Đẳng thức xảy ra khi và chỉ khi a = b = c = 1.
Nhận xét 2.3. Có nhiều công cụ hỗ trợ ta thực hiện phương pháp dồn biến, dưới đây
ta xem xét ứng dụng yếu tố "ít nhất" và bất đẳng thức Cauchy-Schwarz trong việc
giảm biến số của bất đẳng thức. Cụ thể có thể đưa bất đẳng thức ba biến về bất đẳng
17
thức một biến để chứng minh. Ý tưởng của kĩ thuật như sau.
Với bốn số thực a, b, c, k ta có
[(a − k)(b − k)] [(b − k)(c − k)] [(c − k)(a − k)] = (a −k)
2
(b − k)
2
(c − k)
2
≥ 0.
Do đó trong ba số (a −k)(b − k), (b − k)(c − k), (c − k)(a − k) sẽ có "ít nhất"
một số không âm. Giả sử (a − k)(b − k) ≥ 0 thế thì
a
2
+ b

2
= k
2
+ (a + b − k)
2
− 2(a − k)(b − k) ≤ k
2
+ (a + b − k)
2
.
Như vậy để chứng minh bất đẳng thức có giả thiết dạng a + b + c = s và đẳng
thức xảy ra khi hai biến bằng một giá trị nào đó ta có thể sử dụng đánh giá trên để
làm giảm biến số của bất đẳng thức ban đầu. Cụ thể chọn k = m (đảm bảo dấu bằng)
và ta có
a
2
+ b
2
≤ m
2
+ (a + b − m)
2
= m
2
+ (s − c − m)
2
.
Nghĩa là có thể sử dụng đánh giá này vào bài toán thì ta sẽ chỉ còn phải chứng
minh bất đẳng thức của một biến c.
Bài toán 2.18. Với a, b, c là các số thực thỏa mãn điều kiện a + b + c = 3. Chứng

minh rằng
a
7a
2
+ 11
+
b
7b
2
+ 11
+
c
7c
2
+ 11

1
6
.
Lời giải. Bất đẳng thức cần chứng minh
⇔1 −
14a
7a
2
+ 11
+ 1 −
14b
7b
2
+ 11

+ 1 −
14c
7c
2
+ 11
≥ 3 −
14
6

7(a − 1)
2
+ 4
7a
2
+ 11
+
7(b − 1)
2
+ 4
7b
2
+ 11
+
7(c − 1)
2
+ 4
7c
2
+ 11


2
3
⇔4

1
7a
2
+ 11
+
1
7b
2
+ 11
+
1
7c
2
+ 11

+ 7

(a − 1)
2
7a
2
+ 11
+
(b − 1)
2
7b

2
+ 11
+
(c − 1)
2
7c
2
+ 11


2
3
.
Áp dụng bất đẳng thức Cauchy -Schwarz, ta có
1
7a
2
+ 11
+
1
7b
2
+ 11
+
1
7c
2
+ 11

(1 + 1 + 1)

2
7(a
2
+ b
2
+ c
2
) + 33
=
9
7(a
2
+ b
2
+ c
2
) + 33
=
3(a + b + c)
2
21(a
2
+ b
2
+ c
2
) + 11(a + b + c)
2
18
và cũng theo bất đẳng thức Cauchy-Schwarz, ta có

(1 − a)
2
7a
2
+ 11
+
(b − 1)
2
7b
2
+ 11
+
(c − 1)
2
7c
2
+ 11

(1 − a + b − 1 + c − 1)
2
7(a
+
b
2
+ c
2
) + 33
=
(b + c − a − 1)
2

7(a
2
+ b
2
+ c
2
) + 11.3
=
3

b + c − a −
a + b + c
3

2
21(a
2
+ b
2
+ c
2
) + 11(a + b + c)
2
=
4(b + c − 2a)
2
3 [21(a
2
+ b
2

+ c
2
) + 11(a + b + c)
2
]
.
Bất đẳng thức được chứng minh nếu
4.3(a + b + c)
2
21(a
2
+ b
2
+ c
2
) + 11(a + b + c)
2
+
7.4(b + c − 2a)
2
3 [21(a
2
+ b
2
+ c
2
) + 11.(a + b + c)
2
]


2
3
⇔ 2.9(a + b + c)
2
+ 7.2(b + c − 2a)
2
≥ 21(a
2
+ b
2
+ c
2
) + 11(a + b + c)
2
⇔ 42 [a(a − c) − b(a − c)] ≥ 0 ⇔ 42(a − c)(a − b) ≥ 0. (2.6)
Rõ ràng bất đẳng thức trên không phải lúc nào cũng đúng, nhưng từ đó vẫn đi
đến điều phải chứng minh bằng cách đánh giá tương tự
(a − 1)
2
7a
2
+ 11
+
(1 − b)
2
7b
2
+ 11
+
(c − 1)

2
7c
2
+ 11

4(a + c − 2b)
2
3 [21(a
2
+ b
2
+ c
2
) + 11.(a + b + c)
2
]
(a − 1)
2
7a
2
+ 11
+
(b − 1)
2
7b
2
+ 11
+
(1 − c)
2

7c
2
+ 11

4(a + b − 2c)
2
3 [21(a
2
+ b
2
+ c
2
) + 11.(a + b + c)
2
]
và ta lần lượt đưa bài toán về xét tính đúng sai của
42(b − a)(b − c) ≥ 0 (2.7)
42(c − a)(c − b) ≥ 0. (2.8)
mà ta lại có
(a −b)(a −c) + (b −a)(b −c) + (c −a)(c −b) = a
2
+ b
2
+ c
2
−ab −bc −ca ≥ 0.
Do đó trong (2.6), (2.7), (2.8) có ít nhất một bất đẳng thức đúng. Từ đó suy ra
điều phải chứng minh.
Đẳng thức xảy ra khi và chỉ khi a = b = c = 1.
19

Bài toán 2.19. Chứng minh rằng với các số thực dương a, b, c tùy ý ta đều có
(b + c − a)
2
a
2
+ (b + c)
2
+
(c + a − b)
2
b
2
+ (c + a)
2
+
(a + b − c)
2
c
2
+ (a + b)
2

3
5
.
Lời giải. Do bất đẳng thức đã cho thuần nhất với biến a, b, c nên ta hoàn toàn có thể
chuẩn hóa cho a + b + c = 1. Khi đó bất đẳng thức đã cho được viết dưới dạng
(1 − 2a)
2
2a

2
− 2a + 1
+
(1 − 2b)
2
2b
2
− 2b + 1
+
(1 − 2c)
2
2c
2
− 2c + 1

3
5
.
Ta có

a −
1
3

b −
1
3

b −
1

3

c −
1
3

c −
1
3

a −
1
3

=

a −
1
3

2

b −
1
3

2

c −
1

3

2
≥ 0.
Không mất tính tổng quát, giả sử

a −
1
3

b −
1
3

≥ 0.
Ta có
a
2
+ b
2

1
9
+ (a + b −
1
3
)
2
=
1

9
+

2
3
− c

2
.
Áp dụng bất đẳng thức Cauchy-Schwarz, ta có
(1 − 2a)
2
2a
2
− 2a + 1
+
(1 − 2b)
2
2b
2
− 2b + 1

[2 − 2(a + b)]
2
2(a
2
+ b
2
) − 2(a + b) + 2


[2 − 2(a + b)]
2
2

1
9
+ (
2
3
− c)
2

− 2 (1 − c) + 2
=
2c
2
1
9
+

2
3
− c

2
+ c
=
18c
2
9c

2
− 3c + 5
.
Bất đẳng thức được chứng minh nếu ta chứng minh được
(1 − 2c)
2
2c
2
− 2c + 1
+
18c
2
9c
2
− 3c + 5
≥ 35
⇔(3c − 1)
2
(17c
2
− 8c + 5) ≥ 0

luôn đúng

.
Đẳng thức xảy ra khi a = b = c.
20
Bài toán 2.20(China MO 2006). Với các số thực dương a, b, c thỏa mãn a+b+c = 1.
Chứng minh rằng
ab


ab + bc
+
bc

bc + ca
+
ca

ca + ab


2
2
.
Lời giải. Áp dụng bất đẳng thức Cauchy- Schwarz, ta có

ab

ab + bc
+
bc

bc + ca
+
ca

ca + ab

2

=


ab

ab + bc + ca

a(ab + bc + ca)

a + c
+

bc

ab + bc + ca

b(ab + bc + ca)

b + a
+

ca

ab + bc + ca

c(ab + bc + ca)

c + b

2


ab + bc + ca
ab + bc + ca

a(ab + bc + ca)
a + c
+
b(ab + bc + ca)
b + a
+
c(ab + bc + ca)
c + b

= (ab + bc + ca)

a
a + c
+
b
b + a
+
c
c + b

.
Bất đẳng thức được chứng minh nếu ta chứng minh được
(ab + bc + ca)

a
a + c

+
b
b + a
+
c
c + b


1
2
⇔2 (ab + bc + ca)

a
a + c
+
b
b + a
+
c
c + b

≤ (a + b + c)
2
⇔2

a
2
b
a + c
+

ab
2
b + a
+
abc
c + b
+
abc
a + c
+
b
2
c
b + a
+
bc
2
c + b
+
ca
2
a + c
+
abc
a + b
+
c
2
a
c + b


≤ (a + b + c)
2
⇔2

ab + bc + ca +
ab
2
a + b
+
bc
2
b + c
+
ca
2
a + c

≤ (a + b + c)
2
⇔2

ab
2
a + b
+
bc
2
b + c
+

ca
2
a + c

≤ a
2
+ b
2
+ c
2
⇔2b
2

2ab
2
a + b
+ 2c
2

2bc
2
b + c
+ 2a
2

2ca
2
c + a
≥ a
2

+ b
2
+ c
2

2b
3
a + b
+
2c
3
b + c
+
2a
3
c + a
≥ a
2
+ b
2
+ c
2
.

×